Which one of the following could be true?

Seo on August 4, 2020

How do I set up for this game?

I struggled with the set-up for this game, ended up changing it around mid-game, and I still couldn't figure out how to solve the last question of this game: Q 13. I'd very much appreciate a video explanation for set up and the question 13!

Replies
Create a free account to read and take part in forum discussions.

Already have an account? log in

Adamfish on September 11, 2020

Bump - I would also love to see either a video or text explanation of this setup. Thanks!

isabellaivanov on October 6, 2020

Hi! I just did this by eliminating answers knowing that F and G cannot be first and that L has to do the first thing.

a: F has to do H and F cannot be first
b: M can't be second because ghat would mean T would have to be first and L can't do T, F can't be first, and G can't go until F does
c: T can't be first because that would make M be second which we know we can't have (L can't do T)
d: T can't be last because M needs to go right after T
e: W can be first and L would do it!

Nikki.M on April 28, 2021

plz show video of how to set this up, what type of game is this? multilinear ?